Intégrales de juillet

Je vous propose de montrer que:

$\displaystyle \int_0^1 \frac{\ln^3 x}{1-x^2}\ln\left(\frac{1-x}{1+x}\right)\,dx=\frac{93}{16}\zeta(5)-\frac{21}{8}\zeta(2)\zeta(3)$
«1

Réponses

  • Une idée comme ça on pourrait développer en SE la fonction $\frac{1}{1-x^2}\ln(\frac{1-x}{1+x})$
  • Etanche:
    On peut faire comme ça mais c'est sportif. Je trouve ça laid, question de goût personnel.

    Je pense qu'on peut faire ce calcul en utilisant que du calcul intégral et des expressions sous forme d'intégrales de nombres $\zeta(n)$, avec $n\geq 2$ un entier naturel.
  • FDP, que nous laisses-tu supposer comme intégrales connues
    Le 😄 Farceur


  • Bonjour
    tu poses $x = \tanh(t)$ (tangente hyperbolique de $t$) ce qui est possible puisque $- 1 < \tanh(t) < 1.$
    L'intégrale devient $\displaystyle I = \int_0^{+\infty}\ln^3\tanh(t) \ln(e^{-2t})dt, $ soit encore : $$
    I = -2 \int_0^{+\infty}t.\ln^3\tanh(t)dt.
    $$ Intégrale positive puisque le logarithme népérien est négatif.
    L
    'intégrale est d'un abord plus familier sans être évidente.
    Cordialement.
  • JL
    Le problème avec FDP c 'est qu'il part d' une intégrale qu'il sait calculer et complique par des changements de variables successifs pour nous faire surgir un monstre.
    Le 😄 Farceur


  • Une solution consiste a :
    - faire un 1er changement de variable y = (1-x)/(1+x)
    - faire une IPP pour reduire le degre du (Ln)^3
    - faire un 2e changement de variable x = (1-y)/(1+y) pour obtenir x au denominateur

    Apres developpement du (Ln)^2 on est ramene a deux integrales connues :
    - la 1ere est la derivee 4e de B(a,b) en a=o et b=1
    - la 2nde est en 1.10 dans le livre (Almost) Impossible Integrals

    Gebrane a raison : FDP a sciemment complique une integrale.....mais ce fut pour mon grand plaisir.

    fjaclot
  • FJaclot a écrit:
    FDP a sciemment complique une integrale

    En fait, cette intégrale sort d'un calcul qui n'est pas le mien. J'en reparlerai plus tard dès que j'aurai exposé une solution qui n'est pas dans le livre (Almost) Impossible Integrals. (j'y travaille sérieusement malgré la chaleur)
    Pour ce type de calculs je considère que c'est de la "barbarie mathématique" le recours aux séries de Fourier; aux sommes harmoniques (généralisées).
  • Bonjour

    L’intégrale se calcule toute seule avec les poly-logarithmes d’ordres 1,2,3. Je sais qu’on ne voit pas ces fonctions en CM1 mais on peut le faire en Licence.
    On obtient d’ailleurs les intégrales avec un exposant entier arbitraire du logarithme.
  • YvesM:

    J'attends de lire ta solution.
    Cordialement.
  • Je précise que, tous calculs faits, la solution que j’ai indiquée plus haut aboutit au résultat donné par FDP.
  • fjaclot
    Peux-tu donner les détails.
    FDP, on peut écrire ton intégrale comme somme de 4 intégrales de la forme $$\int_0^1\frac{\ln^3(x)\ln(1\pm x)}{(1\pm x)}dx$$
    Laquelle des 4, te pose problème.
    Le 😄 Farceur


  • Gebrane:

    J'ai en effet voulu faire le calcul proposé en calculant ces 4 intégrales.
    $\displaystyle \int_0^1\frac{\ln(1-x)\ln^3 x}{1-x}\,dx$ je l'ai déjà calculée ici.
    Je pense que FJaclot la calcule aussi en utilisant la fonction Bêta en introduisant deux paramètres.
    En fait, les intégrales $\displaystyle \int_0^1\frac{\ln(1-x)\ln^3 x}{1-x}\,dx$ et $\displaystyle \int_0^1\frac{\ln(1+x)\ln^3 x}{1-x}\,dx$
    d'une part, et les intégrales $\displaystyle \int_0^1 \frac{\ln(1-x)\ln^3 x}{1+x}\,dx$, $\displaystyle \frac{\ln(1+x)\ln^3 x}{1+x}\,dx$
    sont liées deux à deux par des relations simples mais ces dernières sont aussi difficiles à obtenir, me semble-t-il, que l'intégrale proposée.
    Donc concrètement je sais calculer les deux premières et pas les deux dernières.
    Mais j'ai changé d'approche, j'essaie d'attaquer directement le problème (je n'ai pas encore fini le calcul entrepris)

    A noter que:

    \begin{align} \frac{1}{8}\int_0^1 \frac{\ln(1-x)\ln^3 x}{1-x}\,dx&=\int_0^1 \frac{2x\ln(1-x^2)\ln^3 x}{1-x^2}\,dx\\
    &=\int_0^1\frac{\ln(1-x^2)\ln^3 x}{1-x}\,dx-\int_0^1\frac{\ln(1-x^2)\ln^3 x}{1+x}\,dx\\
    &=\int_0^1\frac{\ln(1-x)\ln^3 x}{1-x}\,dx+\int_0^1\frac{\ln(1+x)\ln^3 x}{1-x}\,dx-\int_0^1\frac{\ln(1-x)\ln^3 x}{1+x}\,dx-\int_0^1\frac{\ln(1+x)\ln^3 x}{1+x}\,dx
    \end{align}

    PS:

    Si je me souviens bien, en réalité on a seulement besoin de connaître les valeurs des intégrales:
    $\displaystyle \int_0^1\frac{\ln(1-x)\ln^3 x}{1-x}\,dx, \int_0^1\frac{\ln(1+x)\ln^3 x}{1+x}\,dx$
    L'introduction des deux autres s'est faite naturellement du fait, par exemple, de la relation précisée juste au dessus.
  • Bonjour à tous,
    j'ai été curieux de voir ce que cette intégrale pouvait donner en termes de séries, et je trouve
    $\displaystyle \frac{3}{4}\sum_{n=1}^{+\infty}\frac{H(2n)-\tfrac12H(n)}{n^4}$ avec $H(n)$ qui est le $n$-ième nombre harmonique.
    il est curieux de constater que la formule (20) de https://mathworld.wolfram.com/HarmonicNumber.html s'approche de la forme de Fin de partie.
    Cordialement,
    Yalçin
  • Voici où j'ai pêché cette intégrale, elle est baptisée $K$ si j'ai bien lu:

    https://math.stackexchange.com/a/771277/186817


    (dans cette réponse on retrouve la formule donnée par Yalcin)

    J'essaie (toujours) de trouver une solution sans utilisation de séries de Fourier, de séries harmoniques.

    Pour y arriver, une méthode est de calculer les quatre intégrales ci-dessus. A cette heure je sais calculer seulement deux d'entre-elles.
  • Merci Fin de partie pour le lien.

    Avec la même méthode on a
    $$\int_0^1\frac{\ln(1-\varepsilon x)\ln(x)^3}{1-\varepsilon x}\,dx=6\sum_{k=1}^{\infty}\frac{\varepsilon^kH_k}{(k+1)^4},\;\forall\varepsilon\in\{-1,1\}.$$
  • C'est beaucoup plus facile à prouver quand on sait que $\displaystyle -\dfrac{\ln(1-x}{1-x}=H_nx^n$ pour $-1<x<1$.
  • FDP
    peux-tu demander, pour voir,sur ME comment calculer uniquement par ipp et cdv
    $\displaystyle \int_0^1\frac{\ln(1-x)\ln^3 x}{1-x}\,dx, \int_0^1\frac{\ln(1+x)\ln^3 x}{1+x}\,dx$
    Le 😄 Farceur


  • La première je l'ai déjà calculée ici. La deuxième je ne sais pas. Elle me pose problème pour le moment.
  • Bonjour,

    Pour la deuxième, le $1+x$ dans l’argument pose problème, alors que $x,1-x$ ne posent pas problème.

    Tu peux changer les variables $\displaystyle x\leadsto y$ avec $\displaystyle x={y\over 1-y}.$ C’est la méthode générique pour traiter ces intégrales avec les restrictions que tu t’imposes.
  • Bonjour,

    $\displaystyle \int_0^1 {\ln^3 x \ln(1-x) \over 1-x} dx= -6 \zeta(2)\zeta(3) + 12 \zeta(5).$
  • FDP peux-tu donner le lien d'ici pour la première ? et merci
    Le 😄 Farceur


  • Dans le cas d'espèce, $\displaystyle \int_0^1 \frac{\ln(1+x)\ln^3 x }{1+x}\,dx$, j'ai des doutes que les changements de variable $y=\dfrac{x}{1-x}$ ou $y=\dfrac{x}{1+x}$ facilitent le calcul.

    J'avais plutôt pensé à $y=\dfrac{1-x}{1+x}$ mais cela débouche sur des difficultés:

    \begin{align}\int_0^1 \frac{\ln(1+x)\ln^3 x}{1+x}\,dx&\overset{y=\frac{1-x}{1+x}}=\int_0^1 \frac{\ln \left(\frac{2}{1+x}\right)\ln^3\left(\frac{1-x}{1+x}\right)}{1+x}\,dx\\
    &=\int_0^1 \frac{\ln 2\ln^3\left(\frac{1-x}{1+x}\right)}{1+x}\,dx-\int_0^1 \frac{\ln \left(1+x\right)\ln^3\left(\frac{1-x}{1+x}\right)}{1+x}\,dx\\
    &=\ln 2\int_0^1 \frac{\ln^3 x}{1+x}\,dx+\int_0^1 \frac{\ln^4(1+x)}{1+x}\,dx-3\int_0^1 \frac{\ln^3(1+x)\ln(1-x)}{1+x}\,dx+\\
    &3\int_0^1 \frac{\ln^2(1+x)\ln^2(1-x)}{1+x}\,dx-\int_0^1 \frac{\ln(1+x)\ln^3(1-x)}{1+x}\,dx\\
    \end{align}

    Les trois dernières intégrales ne sont pas plus faciles à calculer, a priori (les deux premières le sont).
  • Gebrane:

    Un peu de fraicheur dans cette chaleur estivale:

    http://www.les-mathematiques.net/phorum/read.php?4,1893098,1923136#msg-1923136

    PS:
    On peut essayer de faire la même chose pour les trois intégrales soeur restantes.
    Mais cela ne marche que pour celle que j'ai baptisée $U$.
    Je suis en train de tester des variantes: mais les calculs sont plus délicats.

    PS2:
    En fait le calcul de $U$ est plus simple que le calcul que j'ai mis ci-dessus:

    \begin{align*}
    U&=\int_0^1 \frac{\ln(1+x)\ln^3 x}{1-x}\,dx\\
    U&\overset{\text{IPP}}=\left[\left(\int_0^x \frac{\ln^3 t}{1-t}\,dt\right)\ln(1+x)\right]_0^1-\int_0^1 \frac{1}{1+x}\left(\int_0^x\frac{\ln^3 t}{1-t}\,dt\right)\,dx\\
    &=-6\zeta(4)\ln 2+\int_0^1\int_0^1 \left(\frac{\ln^3(tx)}{(1+t)(1+x)}-\frac{\ln^3(tx)}{(1+t)(1-tx)}\right)\,dt\,dx\\
    &=-6\zeta(4)\ln 2+6\left(\int_0^1\frac{\ln^2 t}{1+t}\,dt\right)\left(\int_0^1\frac{\ln x}{1+x}\,dx\right)+2\left(\int_0^1\frac{\ln^3 t}{1+t}\,dt\right)\left(\int_0^1\frac{1}{1+x}\,dx\right)-\\
    &\int_0^1 \frac{1}{t(1+t)}\left(\int_0^t \frac{\ln^3 u}{1-u}\,du\right)\,dt\\
    &=-\frac{33}{2}\zeta(4)\ln 2-\frac{9}{2}\zeta(2)\zeta(3)-\int_0^1 \frac{1}{t(1+t)}\left(\int_0^t \frac{\ln^3 u}{1-u}\,du\right)\,dt\\
    &\overset{\text{IPP}}=-\frac{33}{2}\zeta(4)\ln 2-\frac{9}{2}\zeta(2)\zeta(3)-\left[\ln\left(\frac{t}{1+t}\right)\left(\int_0^t \frac{\ln^3 u}{1-u}\,du\right)\right]_0^1+\int_0^1 \frac{\ln\left(\frac{t}{1+t}\right)\ln^3 t}{1-t}\,dt\\
    &=-\frac{45}{2}\zeta(4)\ln 2-\frac{9}{2}\zeta(2)\zeta(3)+\int_0^1 \frac{\ln\left(\frac{t}{1+t}\right)\ln^3 t}{1-t}\,dt\\
    &=-\frac{45}{2}\zeta(4)\ln 2-\frac{9}{2}\zeta(2)\zeta(3)+24\zeta(5)-U\\
    U&=\boxed{-\frac{45}{4}\zeta(4)\ln 2-\frac{9}{4}\zeta(2)\zeta(3)+12\zeta(5)}
    \end{align*}
  • Pour que vous vous rendiez compte de ce qui se passe quand la méthode exposée ci-dessus ne fonctionne pas comme attendu:

    \begin{align*}
    V&=\int_0^1 \frac{\ln(1-x)\ln^3 x}{1+x}\,dx,K_2=\int_0^1 \frac{\ln(1+x)\ln^3 x}{1+x}\,dx,C=\int_0^1 \frac{\ln^3 x}{1+x}\,dx\\
    V&\overset{\text{IPP}}=\left[\left(\left(\int_0^x \frac{\ln^3 t}{1+t}\,dt\right)-C\right)\ln(1-x)\right]_0^1+\int_0^1 \frac{1}{1-x}\left(\left(\int_0^x \frac{\ln^3 t}{1+t}\,dt\right)-C\right)\,dx\\
    &=\int_0^1 \frac{1}{1-x}\left(\left(\int_0^x \frac{\ln^3 t}{1+t}\,dt\right)-C\right)\,dx\\
    &=\int_0^1\int_0^1\left(\frac{x\ln^3(tx)}{(1-x)(1+tx)}-\frac{C}{1-x}\right)\,dt\,dx\\
    &=\int_0^1\int_0^1\left(\frac{\ln^3(tx)}{(1-x)(1+t)}-\frac{\ln^3(tx)}{(1+tx)(1+t)}-\frac{C}{1-x}\right)\,dt\,dx\\
    &=\left(\int_0^1 \frac{1}{1+t}dt\right)\left(\int_0^1 \frac{\ln^3 x}{1-x}dx\right)+3\left(\int_0^1 \frac{\ln^2 t}{1+t}dt\right)\left(\int_0^1 \frac{\ln x}{1-x}dx\right)+\\
    &3\left(\int_0^1 \frac{\ln t}{1+t}dt\right)\left(\int_0^1 \frac{\ln^2 x}{1-x}dx\right)+\int_0^1\int_0^1\left(\frac{\ln^3 t}{(1-x)(1+t)}-\frac{\ln^3(tx)}{(1+tx)(1+t)}-\frac{C}{1-x}\right)dtdx\\
    &=-\frac{15}{2}\zeta(2)\zeta(3)-6\zeta(4)\ln 2+\int_0^1\int_0^1\left(\frac{\ln^3 t}{(1-x)(1+t)}-\frac{\ln^3(tx)}{(1+tx)(1+t)}-\frac{C}{1-x}\right)dtdx\\
    &0\leq A <1\\
    V(A)&=-\frac{15}{2}\zeta(2)\zeta(3)-6\zeta(4)\ln 2+\int_0^1 \left(\int_0^a \left( \frac{\ln^3 t}{(1-x)(1+t)}-\frac{\ln^3(tx)}{(1+tx)(1+t)}-\frac{C}{1-x}\right)dx\right)dt\\
    &=-\frac{15}{2}\zeta(2)\zeta(3)-6\zeta(4)\ln 2+\ln(1-a)\left(C-\int_0^a \frac{\ln^3 t}{1+t}dt\right)-\int_0^1 \left(\int_0^a \frac{\ln^3(tx)}{1+tx}dx\right)dt\\
    V&=\lim_{V\rightarrow 1}V(A)\\
    &\overset{u(x)=tx}=-\frac{15}{2}\zeta(2)\zeta(3)-6\zeta(4)\ln 2-\int_0^1 \frac{1}{t(1+t)}\left(\int_0^t \frac{\ln^3 u}{1+u}\,du\right)\,dt\\
    &\overset{\text{IPP}}=-\frac{15}{2}\zeta(2)\zeta(3)-6\zeta(4)\ln 2+\left[\ln\left(\frac{1+t}{t}\right)\left(\int_0^t \frac{\ln^3 u}{1+u}\,du\right)\right]_0^1+\int_0^1 \frac{\ln\left(\frac{t}{1+t}\right)\ln^3 t}{1+t}\,dt\\
    &=\frac{45}{2}\zeta(5)-\frac{15}{2}\zeta(2)\zeta(3)-\frac{45}{4}\zeta(4)\ln 2-K_2\\
    \end{align*}

    NB:
    Dans tout ce qui précède j'ai utilisé les valeurs connues des intégrales du type $\displaystyle \int_0^1 \frac{\ln^m x}{1+x}\,dx$ ou $\displaystyle \int_0^1 \frac{\ln^m x}{1-x}\,dx$, avec $m$ entier naturel non nul.
  • Pour le calcul de la dernière intégrale, $K_2$:

    \begin{align*}
    K_2&=\int_0^1 \frac{\ln(1+x)\ln^3 x}{1+x}\,dx\\
    D&=\int_0^1 \frac{\ln^3 x}{1+x}\,dx,
    F=\int_0^1 \frac{\ln^3 x}{1-x}\,dx\\
    U&=\int_0^1 \frac{\ln(1+x)\ln^3 x}{1-x}\,dx,V=\int_0^1 \frac{\ln(1-x)\ln^3 x}{1+x}\,dx\\
    K_2&\overset{\text{IBP}}=\left[\left(\int_0^x\frac{\ln^3 t}{1+t}\right)\ln(1+x)\right]_0^1-\int_0^1 \frac{1}{1+x}\left(\int_0^x\frac{\ln^3 t}{1+t}\right)\,dx\\
    &=D\ln 2-\int_0^1 \int_0^1 \frac{x\ln^3(tx)}{(1+x)(1+tx)}\,dt\,dx\\
    &=D\ln 2+\int_0^1 \int_0^1 \left(\frac{\ln^3(tx)}{(1-t)(1+x)}-\int_0^1 \frac{\ln^3(tx)}{(1-t)(1+tx)}\right)\,dt\,dx\\
    &=D\ln 2+\left(\int_0^1 \frac{\ln^3 t}{1-t}\,dt\right)\left(\int_0^1 \frac{1}{1+x}\,dx\right)+3\left(\int_0^1 \frac{\ln^2 t}{1-t}\,dt\right)\left(\int_0^1 \frac{\ln x}{1+x}\,dx\right)+\\
    &3\left(\int_0^1 \frac{\ln t}{1-t}\,dt\right)\left(\int_0^1 \frac{\ln^2 x}{1+x}\,dx\right)+\int_0^1 \left(\frac{D}{1-t}-\frac{1}{1-t}\left(\int_0^1 \frac{\ln^3 (tx)}{1+tx}\,dx\right)\right)\,dt\\
    &=D\ln 2+F\ln 2-\frac{15}{2}\zeta(2)\zeta(3)+\int_0^1 \left(\frac{D}{1-t}-\frac{1}{t(1-t)}\left(\int_0^t \frac{\ln^3 u}{1+u}\,du\right)\right)\,dt\\
    &0\leq A <1\\
    K_2(A)&=D\ln 2+F\ln 2-\frac{15}{2}\zeta(2)\zeta(3)+\int_0^A \left(\frac{D}{1-t}-\frac{1}{t(1-t)}\left(\int_0^t \frac{\ln^3 u}{1+u}\,du\right)\right)\,dt\\
    &=D\Big(\ln 2-\ln(1-A)\Big)+F\ln 2-\frac{15}{2}\zeta(2)\zeta(3)-\int_0^A \frac{1}{t(1-t)}\left(\int_0^t \frac{\ln^3 u}{1+u}\,du\right)\,dt\\
    &=D\Big(\ln 2-\ln(1-A)\Big)+F\ln 2-\frac{15}{2}\zeta(2)\zeta(3)+\left[\ln\left(\frac{1-t}{t}\right)\left(\int_0^t \frac{\ln^3 u}{1+u}\,du\right)\right]_0^A+\\
    &\int_0^A\frac{\ln\left(\frac{t}{1-t}\right)\ln^3 t}{1+t}\,dt\\
    &=D\ln 2+\ln(1-A)\left(\int_0^A\frac{\ln^3 u}{1+u}\,du-D\right)-\ln A\int_0^A\frac{\ln^3 u}{1+u}\,du+F\ln 2-\frac{15}{2}\zeta(2)\zeta(3)+\\
    &\int_0^A\frac{\ln^4 t}{1+t}\,dt-\int_0^A\frac{\ln\left(1-t\right)\ln^3 t}{1+t}\,dt\\
    K_2&=\lim_{A\rightarrow 1}K_2(A)\\
    &=D\ln 2+F\ln 2-\frac{15}{2}\zeta(2)\zeta(3)+\int_0^1\frac{\ln^4 t}{1+t}\,dt-V\\
    &=-\frac{45}{4}\zeta(4)\ln 2-\frac{15}{2}\zeta(2)\zeta(3)+\frac{45}{2}\zeta(5)-V\\
    K_2+V&=-\frac{45}{4}\zeta(4)\ln 2-\frac{15}{2}\zeta(2)\zeta(3)+\frac{45}{2}\zeta(5)\\
    \end{align*}

    Hélas cela redonne une relation déjà obtenue.
  • La relation obtenue deux fois par des calculs assez fastidieux s'obtient plus facilement de la sorte:

    \begin{align}
    K_1&=\int_0^1 \frac{\ln(1-x)\ln^3 x}{1-x}\,dx,K2=\int_0^1 \frac{\ln(1+x)\ln^3 x}{1+x}\,dx\\
    U&=\int_0^1 \frac{\ln(1+x)\ln^3 x}{1-x}\,dx,V=\int_0^1 \frac{\ln(1-x)\ln^3 x}{1+x}\,dx\\
    K_1+U-K_2-V&=\int_0^1 \frac{\ln(1-x^2)\ln^3 x}{1-x}\,dx-\int_0^1 \frac{\ln(1-x^2)\ln^3 x}{1+x}\,dx\\
    &=\int_0^1 \frac{2x\ln(1-x^2)\ln^3 x}{1-x^2}\,dx\\
    &\overset{y=x^2}=\frac{1}{8}K_1\\
    \end{align}

    En définitive, pour le moment je ne sais calculer que $U,K_1$.
  • Bonjour,

    Si ça aide, avec tes notations :
    $\displaystyle K_1 = \int_0^1 {\ln^3 x \ln(1-x) \over 1-x} dx= -6 \zeta(2) \zeta(3) + 12 \zeta(5)$
    $\displaystyle U = \int_0^1 {\ln^3 x \ln(1+x) \over 1-x} dx= -\frac94 \zeta(2) \zeta(3)-{45 \over 4} \zeta(4) \ln 2 + 12 \zeta(5)$
    $\displaystyle K_2 = \int_0^1 {\ln^3 x \ln(1+x) \over 1+x} dx= -3 \zeta(2) \zeta(3)+{87 \over 16 }\zeta(5)$
    $\displaystyle V = \int_0^1 {\ln^3 x \ln(1-x) \over 1+x} dx = -\frac92 \zeta(2) \zeta(3)-{45 \over 4} \zeta(4) \ln 2+{273 \over 16 }\zeta(5)$
    et
    $\displaystyle I = \int_0^1 {\ln^3 x \over 1-x^2} \ln{1-x \over 1+x}dx={1 \over 2} (K_1-U+V-K_2) =-{21 \over 8} \zeta(2) \zeta(3) + {93 \over 16} \zeta(5).$
  • Bonjour YvesM,
    très jolies formules,
    peux-tu donner tes idées pour le calcul de $K_2$ ?
    merci bien,
    Y.
  • Bonjour,

    Je reste vague exprès.

    D’abord une série avec les nombres harmoniques. Puis pour tout $n$ entier non nul, $\displaystyle H_n=\int_0^1 {1-x^n\over 1-x} dx $ et on permute série et intégrale. On tombe sur du poly logarithme d’ordre $4$ avec comme argument $x$ et $x^2.$
    On utilise les propriétés des poly logarithmes d’arguments $x^2$ pour transformer en $x$ et $-x.$

    Et là, c’est chaud : intégrations par parties dans tous les sens. Trois pages de calculs à la con. Et on trouve.

    Je ne conseille pas de s’aventurer sur ce chemin. Il faut bien connaître les poly logarithmes et avoir du temps à perdre. On n’apprend rien de bon.

    Mais c’est la seule façon que j’ai trouvée. Je pense qu’on peut faire bien mieux avec des fonctions bêtas et hyper géométrique $_2 F_1$ mais je n’ai pas le temps de regarder.

    Wolfram en ligne ne donne pas les résultats, mais cette version est bridée. Il faudrait qu’on pro fasse tourner Wolfram... ça devrait passer.
  • merci beaucoup YvesM (tu)
    Y.
  • Bonjour voici ma méthode.106638
  • Espacepro: merci pour ta contribution mais c'est illisible et je n'ai pas la patience d'un Champollion pour le déchiffrement des hiéroglyphes.
  • Tu m'as rappelé cependant que $x\rightarrow \dfrac{\ln\left(\frac{1-x}{1+x}\right)}{1-x^2}$ avait une primitive élémentaire.
    Je t'en remercie.


    PS:
    On a:
    $\displaystyle \int \dfrac{\ln\left(\frac{1-x}{1+x}\right)}{1-x^2}\,dx=-\frac{1}{4}\ln^2\left(\frac{1-x}{1+x}\right)$

    PS2:
    L'utilisation de cette primitive (donc une intégration par parties) me semble conduire à obtenir une relation entre les 4 intégrales déjà mentionnées. Mais il y a gros à parier que c'est une relation que j'ai déjà obtenue malheureusement.
  • Comme pressenti cela n'apporte aucun renseignement supplémentaire mais le calcul est intéressant tout de même:

    \begin{align*}
    J&=\int_0^1 \frac{\ln\left(\frac{1-x}{1+x}\right)\ln^3 x}{1-x^2}\,dx\\
    &\overset{\text{IPP}}=\left[-\frac{1}{4}\ln^2\left(\frac{1-x}{1+x}\right)\ln^3 x\right]_0^1+\frac{3}{4}\int_0^1 \frac{\ln^2\left(\frac{1-x}{1+x}\right)\ln^2 x}{x}\,dx\\
    &=\frac{3}{4}\int_0^1 \frac{\ln^2\left(\frac{1-x}{1+x}\right)\ln^2 x}{x}\,dx\\
    &=\frac{3}{2}\int_0^1 \frac{\big(\ln^2(1-x)+\ln^2(1+x)\big)\ln^2 x}{x}\,dx-\frac{3}{4}\int_0^1 \frac{\ln^2(1-x^2)\ln^2 x}{x}\,dx\\
    &\overset{\text{IPP}}=\frac{1}{2}\Big[\big(\ln^2(1-x)+\ln^2(1+x)\big)\ln^3 x\Big]_0^1-\int_0^1 \left(\frac{\ln(1+x)}{1+x}-\frac{\ln(1-x)}{1-x}\right)\ln^3 x dx-\\
    &\frac{3}{4}\int_0^1 \frac{\ln^2(1-x^2)\ln^2 x}{x}\,dx\\
    &=K_1-K_2-\frac{3}{4}\int_0^1 \frac{\ln^2(1-x^2)\ln^2 x}{x}\,dx\\
    &\overset{u=x^2}=K_1-K_2-\frac{3}{32}\int_0^1 \frac{\ln^2(1-u)\ln^2 u}{u}\,du\\
    &\overset{\text{IPP}}=K_1-K_2-\frac{1}{32}\Big[\ln^2(1-u)\ln^3 u\Big]_0^2-\frac{1}{16}\int_0^1 \frac{\ln(1-u)\ln^3 u}{1-u}\,dx\\
    &=\frac{15}{16}K_1-K_2
    \end{align*}

    Or, on sait que $\displaystyle J=\frac{1}{2}\Big(K_1-U+V-K_2\Big)$

    Ces deux égalités vont permettre d'avoir une expression de $K_2+V$ en fonction seulement de $U,K_1$
    mais on avait déjà une telle expression.
  • Bonjour FDP
    Peux-tu mettre ta solution sur ME pour le calcul de $\int_0^1\frac{\ln^3x\ln(1-x)}{1-x}dx$
    Shather dit qu'il ne sait pas
    Le 😄 Farceur


  • Gebrane:
    Volontiers mais quel est le message?
  • Gebrane:
    Merci. J'ai des réticences à poser des questions dont j'ai déjà la réponse.
    Pour le moment je n'ai pas de réponse à la question posée (autres que ce qui est déjà proposé)
  • FDP ma question porte sur les 4 intégrales, tu peux mettre ta solution sur le calcul de $\int_0^1\frac{\ln^3x\ln(1-x)}{1-x}dx$
    Le 😄 Farceur


  • Bonjour gebrane,
    ici se trouve une réponse à ta question,
    cordialement,
    Y.
  • Bonjour Yalcin Je n' y vois rien et je n'ose pas te demander de réécrire clairement ce que tu vois dans le scan
    Le 😄 Farceur


  • Les trucs de Sioux mathématiciens avec des intégrales doubles ne sont pas suffisants pour calculer $K_2$ (et $V$)
    Il faut introduire l'intégrale $\displaystyle \int_0^{\frac{1}{2}}\frac{\ln^3 x\ln(1-x)}{1-x}\,dx$ (ce qui revient à considérer des polylogarithmes avec l'argument $\dfrac{1}{2}$)
    J'avais déjà fait des calculs similaires, me semble-t-il, pour calculer $\displaystyle \int_0^{1}\frac{\ln^2 x\ln(1+x)}{1+x}\,dx$. Je pense que j'ai rempli le but que je m'étais assigné (calcul de $K_1,K_2,U,V$ sans utilisation de séries de Fourier, de séries harmoniques)


    \begin{align*}
    C_1&=\int_0^{\frac{1}{2}}\frac{\ln^4 x}{1-x}\,dx,K_3=\int_0^{\frac{1}{2}}\frac{\ln^3 x\ln(1-x)}{1-x}\,dx\\
    \int_0^1 \frac{\ln^4\left(\frac{x}{1+x}\right)}{1+x}\,dx&=\int_0^1 \frac{\ln^4\left(1+x\right)}{1+x}\,dx-4\int_0^1 \frac{\ln^3\left(1+x\right)\ln x}{1+x}\,dx+6\int_0^1\frac{\ln^2\left(1+x\right)\ln^2 x}{1+x}\,dx-\\
    &4\int_0^1 \frac{\ln\left(1+x\right)\ln^3 x}{1+x}\,dx+\int_0^1 \frac{\ln^4 x}{1+x}\,dx\\
    &=\frac{1}{5}\ln^5 2-\Big[\ln^4(1+x)\ln x\Big]_0^1+\int_0^1 \frac{\ln^4(1+x)}{x}\,dx+\\
    &2\Big[\ln^3(1+x)\ln^2 x\Big]_0^1-4\int_0^1 \frac{\ln^3(1+x)\ln x}{x}\,dx-4K_1+\frac{45}{2}\zeta(5)\\
    &=\frac{1}{5}\ln^5 2-4\int_0^1 \frac{\ln^3(1+x)\ln x}{x}\,dx+\int_0^1 \frac{\ln^4(1+x)}{x}\,dx-4K_2+\frac{45}{2}\zeta(5)\\
    &\overset{y=\frac{1}{1+x}}=\frac{1}{5}\ln^5 2-4\int_{\frac{1}{2}}^1\frac{\ln^3 y\Big(\ln y-\ln(1-y)\Big)}{y(1-y)}\,dy+\int_{\frac{1}{2}}^1\frac{\ln^4 y}{y(1-y)}\,dy-4K_2+\frac{45}{2}\zeta(5)\\
    &=\frac{1}{5}\ln^5 2-3\int_{\frac{1}{2}}^1\frac{\ln^4 x}{x}\,dx-3\int_{\frac{1}{2}}^1\frac{\ln^4 x}{1-x}\,dx+4\int_{\frac{1}{2}}^1\frac{\ln^3 x\ln(1-x)}{x}\,dx+\\
    &4\int_{\frac{1}{2}}^1\frac{\ln^3 x\ln(1-x)}{1-x}\,dx-4K_2+\frac{45}{2}\zeta(5)\\
    &=-\frac{2}{5}\ln^5 2-3\Big(24\zeta(5)-C_1\Big)+4\int_{\frac{1}{2}}^1\frac{\ln^3 x\ln(1-x)}{x}\,dx+4\Big(K_1-K3\Big)-4K_2+\\
    &\frac{45}{2}\zeta(5)\\
    &\overset{\text{IPP}}=-\frac{2}{5}\ln^5 2-3\Big(24\zeta(5)-C_1\Big)+\Big[\ln^4 x\ln(1-x)\Big]_{\frac{1}{2}}^1+\int_{\frac{1}{2}}^1\frac{\ln^4 x}{1-x}\,dx+4\Big(K_1-K3\Big)-\\
    &4K_2+\frac{45}{2}\zeta(5)\\
    &=\frac{3}{5}\ln^5 2-2\Big(24\zeta(5)-C_1\Big)+4\Big(K_1-K3\Big)-4K_2+\frac{45}{2}\zeta(5)\\
    \end{align*}
    Par ailleurs,

    \begin{align*}
    \int_0^1 \frac{\ln^4\left(\frac{x}{1+x}\right)}{1+x}\,dx&\overset{y=\frac{x}{1+x}}=C_1
    \end{align*}
    Donc,
    \begin{align*}
    C_1&=\frac{3}{5}\ln^5 2-2\Big(24\zeta(5)-C_1\Big)+4\Big(K_1-K3\Big)-4K_2+\frac{45}{2}\zeta(5)\\
    K_2+K_3&=\frac{3}{20}\ln^5 2+\frac{1}{4}C_1+K_1-\frac{51}{8}\zeta(5)
    \end{align*}
    En outre,
    \begin{align*}
    K_2&\overset{y=\frac{1}{1+x}}=\int_{\frac{1}{2}}^1\frac{\ln y\Big(\ln y-\ln(1+y)\Big)^3}{y}\,dy\\
    &=\int_{\frac{1}{2}}^1\left(\frac{\ln^4 y}{y}-\frac{3\ln(1-y)\ln^3 y}{y}+\frac{3\ln^2(1-y)\ln^2 y}{y}-\frac{\ln^3(1-y)\ln y}{y}\right)\,dy\\
    &\overset{\text{IPP}}=\frac{1}{5}\ln^5 2-\frac{3}{4}\Big[\ln^4 y\ln(1-y)\Big]_{\frac{1}{2}}^1-\frac{3}{4}\int_{\frac{1}{2}}^1 \frac{\ln^4 y}{1-y}\,dy+\Big[\ln^3 y\ln^2(1-y)\Big]_{\frac{1}{2}}^1+\\
    &2\int_{\frac{1}{2}}^1 \frac{\ln^3 y\ln(1-y)}{1-y}\,dy-\int_{\frac{1}{2}}^1 \frac{\ln^3(1-y)\ln y}{y}\,dy\\
    &=\frac{9}{20}\ln^5 2-\frac{3}{4}\Big(24\zeta(5)-C1\Big)+2\Big(K_1-K3\Big)-\int_{\frac{1}{2}}^1 \frac{\ln^3(1-y)\ln y}{y}\,dy\\
    &\overset{z=1-y}=\frac{9}{20}\ln^5 2-\frac{3}{4}\Big(24\zeta(5)-C1\Big)+2\Big(K_1-K3\Big)-K3\\
    K_2+3K_3&=\frac{9}{20}\ln^5 2-18\zeta(5)+\frac{3}{4}C_1+2K_1\\
    \end{align*}
    Donc,
    \begin{align*}
    K_2&=\frac{1}{2}K_1-\frac{9}{16}\zeta(5)\\
    &=\frac{1}{2}\Big(12\zeta(5)-6\zeta(2)\zeta(3)\Big)-\frac{9}{16}\zeta(5)\\
    \end{align*}
    Ainsi,
    $\boxed{K_2=\displaystyle \frac{87}{16}\zeta(5)-3\zeta(2)\zeta(3)}$

    PS:

    On peut calculer $\displaystyle \int_0^{\frac{1}{2}}\frac{\ln^3 x\ln(1-x)}{1-x}\,dx$ mais cela suppose de calculer $C_1$.
    Ce dernier calcul est relativement aisé et on va se retrouver avec des polylogarithmes avec l'argument $\dfrac{1}{2}$
  • Très bizarre, quand j'ai cliqué sur le lien de Yalcin, il y a quelques jours, son lien m'a envoyé vers le scan de espace pro. Ce matin j'ai cliqué et ça m'envoie vers MSE.
    Le 😄 Farceur


  • Gebrane:

    Tu m'as arnaqué B-)-

    Tu as refilé les points sur M.E à une réponse qui utilise de la dérivation sous le signe intégral*. :-D
    (Ne change rien, ce qui est donné est donné...)

    *: Dans l'utilisation de la fonction Bêta on introduit un paramètre qui va servir de variable pour dériver sous le signe intégral.
  • j'avais une impression que tu n y arriverais pas dans la semaine, j'ai préféré accepter
    Le 😄 Farceur


  • J'ai exploré beaucoup d'impasses puis je me suis souvenu de ce que j'avais fait précédemment et cela a fonctionné.
    Il ne faut pas enterrer trop vite mézigue B-)-
  • Bonjour,

    @FDP, quand je t'ai indiqué d'utiliser le changement de variables tu as écrit 'j'ai des doutes que les changements de variables $\displaystyle y={x \over 1-x}$ ou $\displaystyle y={x \over 1+x}$ facilitent le calcul.'

    Effectivement, mon indication donne la borne $1/2$ en $x=1$ qui semble contre productive mais qui est bien le moyen générique de traiter les intégrales avec $\displaystyle \ln(1+x).$

    J'espère qu'on va passer à d'autres intégrales... il me semble qu'on a fait le tour des numérateurs en $\ln x, \ln (1-x), \ln(1+x), \ln(1+x^2)$ et des dénominateurs en $x, 1-x, 1+x, 1+x^2.$
  • YvesM:

    J'attends de voir ta solution.
    Comme aux échecs on croit qu'on a la bonne séquence de coups qui va conclure la partie à notre avantage mais malheureusement on a mal évalué les conséquences d'un coup de la séquence et finalement la partie est perdue.

    Note bien que j'ai suivi à la lettre ton conseil pour calculer $K_2,V$.

    Voilà ce que cela donne. On se retrouve à calculer:

    \begin{align*}
    W&=\int_0^1 \frac{\ln(2-x)\ln^3 x}{2-x}\,dx,H=\int_0^1 \frac{\ln^3 x}{2-x}\,dx\\
    &\overset{\text{IPP}}=\left[\left(\int_0^x \frac{\ln^3 t}{2-t}\,dt\right)\ln(2-x)\right]_0^1+\int_0^1 \frac{1}{2-x}\left(\int_0^x \frac{\ln^3 t}{2-t}\,dt\right)\,dx\\
    &=\int_0^1 \frac{1}{2-x}\left(\int_0^x \frac{\ln^3 t}{2-t}\,dt\right)\,dx\\
    &=\int_0^1 \int_0^1\frac{x\ln^3(tx)}{(2-x)(2-tx)}\,dt\,dx\\
    &=\int_0^1 \int_0^1\left(\frac{\ln^3(tx)}{(1-t)(2-x)}-\frac{\ln^3(tx)}{(1-t)(2-tx)}\right)\,dt\,dx\\
    &=\left(\int_0^1 \frac{\ln^3 t}{1-t}\,dt\right)\left(\int_0^1 \frac{1}{2-x}\,dx\right)+3\left(\int_0^1 \frac{\ln t}{1-t}\,dt\right) \left(\int_0^1\frac{\ln^2 x}{2-x}\,dx\right)+\\
    &+3\left(\int_0^1 \frac{\ln^2 t}{1-t}\,dt\right) \left(\int_0^1\frac{\ln x}{2-x}\,dx\right)+\int_0^1 \int_0^1\left(\frac{\ln^3 x}{(1-t)(2-x)}-\frac{\ln^3(tx)}{(1-t)(2-tx)}\right)\,dt\,dx\\
    &=-6\zeta(4)\ln 2-3\zeta(2)\left(\frac{7}{4}\zeta(3)-\zeta(2)\ln 2+\frac{1}{3}\ln^3 2\right)+\frac{3}{2}\Big(\ln^2 2-\zeta(2)\Big)\times 2\zeta(3)+\\
    &\int_0^1 \int_0^1\left(\frac{\ln^3 x}{(1-t)(2-x)}-\frac{\ln^3(tx)}{(1-t)(2-tx)}\right)\,dt\,dx\\
    &=-6\zeta(4)\ln 2-\zeta(2)\ln^3 2+3\zeta(3)\ln^2 2+3\zeta(2)^2\ln 2-\frac{33}{4}\zeta(2)\zeta(3)+\\
    &\int_0^1 \int_0^1\left(\frac{\ln^3 x}{(1-t)(2-x)}-\frac{\ln^3(tx)}{(1-t)(2-tx)}\right)\,dt\,dx\\
    &\int_0^1 \int_0^1\left(\frac{\ln^3 x}{(1-t)(2-x)}-\frac{\ln^3(tx)}{(1-t)(2-tx)}\right)\,dt\,dx\\
    &=-6\zeta(4)\ln 2-\zeta(2)\ln^3 2+3\zeta(3)\ln^2 2+3\zeta(2)^2\ln 2-\frac{33}{4}\zeta(2)\zeta(3)+\\
    &\int_0^1 \left(\frac{H}{1-t}-\frac{1}{t(1-t)}\left(\int_0^t \frac{\ln^3 u}{2-u}\,du\right)\right)\,dt\\
    &0\leq A<1\\
    W(A)&=-6\zeta(4)\ln 2-\zeta(2)\ln^3 2+3\zeta(3)\ln^2 2+3\zeta(2)^2\ln 2-\frac{33}{4}\zeta(2)\zeta(3)+\\
    &\int_0^A \left(\frac{H}{1-t}-\frac{1}{t(1-t)}\left(\int_0^t \frac{\ln^3 u}{2-u}\,du\right)\right)\,dt\\
    &\overset{\text{IPP}}=-6\zeta(4)\ln 2-\zeta(2)\ln^3 2+3\zeta(3)\ln^2 2+3\zeta(2)^2\ln 2-\frac{33}{4}\zeta(2)\zeta(3)+\\
    &-H\ln(1-A)+\left[\ln\left(\frac{1-t}{t}\right)\left(\int_0^t \frac{\ln^3 u}{2-u}\,du\right)\right]_0^A+\int_0^A\frac{\ln\left(\frac{1-t}{t}\right)\ln^3 t}{2-t}dt\\
    \end{align*}

    Je n'ai pas continué car on se retrouve avec sur les bras l'intégrale: $\displaystyle \int_0^1\frac{\ln\left(1-t\right)\ln^3 t}{2-t}dt$ qu'il me semble difficile d'exprimer simplement en fonction de $U,V,K_1,K_2$.

    J'aimerais bien voir comment à coups de fonction Bêta et/ou de séries harmoniques certains tenteraient de calculer cette dernière intégrale. B-)-

    Avec ce qui précède (dans la file de messages) on doit certainement pouvoir montrer que:
    $\displaystyle \int_0^1\frac{\ln\left(1-t\right)\ln^3 t}{2-t}dt=-\frac{3}{16}\zeta(5)-\frac{21}{4}\zeta(2)\zeta(3)+\frac{9}{4}\zeta(4)\ln 2-\frac{3}{20}\ln^5 2+18 \text{Li}_5\left(\frac{1}{2}\right)$
    (je n'ai pas réellement fait ce calcul)
  • Pour revenir à ma marotte.
    Je vous propose de montrer que $\displaystyle \zeta(4)=\dfrac{2}{5}\zeta(2)^2$

    Mais uniquement avec les outils du calcul intégral c'est à dire qu'on s'interdit l'usage de séries de Fourier et de toutes autres séries.
    On s'autorise seulement une représentation sous forme d'intégrale des nombres $\zeta(2),\zeta(4)$.

    PS:
    Je n'ai pas encore finalisé totalement ce calcul mais je pense que c'est possible avec les restrictions indiquées.
Connectez-vous ou Inscrivez-vous pour répondre.